1
$\begingroup$

I am given a question such that a 0.280kg object has a displacement (in meters) of $x=5t^3-8t^2-30t$. I need to find the average net power input from the interval of $t=2.0s$ to $t=4.0s$.

I know the formula for average net power is $\frac{\int^{x_2}_{x_1}F \ dx}{t_2-t_1}$ as the force (acceleration is not constant). The acceleration is given by $a = 30t-16$. The force is then given by $F=ma$, but since $m$ is a constant, I intend to ignore it in my calculations. As such, all I need to do is to express acceleration as a function of displacement $x$.

I initially tried to substitute $t=\frac{a+16}{30}$ into the displacement equation, but ended up with a complex expression in $a$ that I could not integrate $x$ against.

I then attempted to try chain rule, with $\frac{da}{dx} = \frac{da}{dt} \div\frac{dx}{dt} =\frac{30}{15t^2-16t-30}$, but this is an expression in $t$ and I still cannot perform $\int^{x_2}_{x_1} F \ dx$.

Does anyone have any advice on what I can do? Many thanks for any help extended!

$\endgroup$
1
  • 3
    $\begingroup$ Try this $Fdx=F\dfrac{dx}{dt}dt=mavdt$ $\endgroup$
    – Eli
    Commented Feb 17, 2022 at 13:00

1 Answer 1

1
$\begingroup$

You can use the fact that $\frac{dx}{dt} = 15t^{2} -16t -30$ first of all. Then, you can make a substitution into your work done integral, for $dt$, and change the limits so that instead of the displacement $x_{i}$, you have whatever initial and final times $t_{i}$. That should then work! If not, let me know and I can give you an explicit answer, but do try it yourself first.

$\endgroup$
3
  • 1
    $\begingroup$ Thank you so much. I kinda get what you mean, essentially I will replace $dx$ with $dx=15t^2-16t-30 \ dt$. In the replacement of the limits, does that mean I can simply put in $t=2$ and $t=4$? Is my understanding correct? $\endgroup$
    – a9302c
    Commented Feb 17, 2022 at 13:12
  • $\begingroup$ Yep, exactly right! If you're happy with my response you can accept it as answer :) $\endgroup$
    – jambajuice
    Commented Feb 17, 2022 at 14:11
  • 1
    $\begingroup$ Done! Thank you so much for the assistance! $\endgroup$
    – a9302c
    Commented Feb 18, 2022 at 3:33

Your Answer

By clicking “Post Your Answer”, you agree to our terms of service and acknowledge you have read our privacy policy.

Not the answer you're looking for? Browse other questions tagged or ask your own question.